Actual CAT Problems 1998-2008 Algebra (Answers and Explanations)

You might also like

Download as pdf or txt
Download as pdf or txt
You are on page 1of 15

Actual CAT Problems 1998-2008

Algebra (Answers and Explanations)


CAT 1999
1. a

The difference between two integers will be 1, only if


one is even and the other one is odd. 4x will always
be even, so 17y has to be odd and hence y has to be
odd.
Moreover, the number 17y should be such a number
that is 1 less than a multiple of 4. In other words, we
have to find all such multiples of 17, which are 1 less
than a multiple of 4. The first such multiple is 51. Now
you will find that as the multiples of 17 goes on
increasing, the difference between it and its closest
higher multiple of 4 is in the following pattern, 0, 3, 2, 1,
e.g. 52 51 = 1,
68 68 = 0, 88 85 = 3, 104 102 = 2, 120 119 = 1,
136 136 = 0
So the multiples of 17 that we are interested in are 3,
7, 11, 15 .
Now since, x 1000, 4x 4000 . The multiple of 17
closest and less than 4000 is 3995 (17 235). And
incidentally, 3996 is a multiple of 4, i.e. the difference
is 4.
This means that in order to find the answer, we need
to find the number of terms in the AP formed by
3, 7, 11, 15 235, where a = 3, d = 4.
Since, we know that Tn = a + (n 1)d,
so 235 = 3 + (n 1) 4. Hence, n = 59.

or (r 6) = 11, r = 5
2q 12=8 2q 12 = 8 , q = 10
or 2q 12 = 8 , q =2
Hence, minimum value of

q
10
=
= 2.
r
5

For questions 4 to 6:

Place of
w orship

Number of
flow ers
before
offering

Number of
flow ers
offered

Number of
flow ers
left

(15/8)y

(7/8)y

(7/4) y

(3/4)y

(3/2) y

y/2

Starting from the fourth place of worship and moving


backwards, we find that number of flowers before

15
y.
8

Alternate Solution:
4x 17y = 1 and x 1000

entering the first place of worship is

so, 17y + 1 4000 i.e. y 235 and moreover every 4th


value of y with give value of x.

Hence, number of flowers before doubling =

235
= 58
So, number of values =
4
So, total number of terms will be 58 + 1 = 59
2. a

r 6 = 11 r 6 = 11, r = 17

3. d

Let x be the fixed cost and y the variable cost


17500 = x + 25y
(i)
30000 = x + 50y
(ii)
Solving the equation (i) and (ii), we get
x = 5000, y = 500
Now if the average expense of 100 boarders be A.
Then
100 A = 5000 + 500 100
A = 550.

Algebra

15
y
16

(but this is equal to 30)


Hence, y = 32
Answer for 4 is (c)
The minimum value of y so that

15
y is a whole number
16

is 16.
Therefore, 16 is the minimum number of flowers that
can be offered.
Answer for 5 is (c).

15
y = 15 .
16
Hence, the minimum number of flowers with which
Roopa leaves home is 15.
Answer for6 is (b).

For y = 16, the value of

Page 1

7. d

8. b

Let m = 1. So, option (a) will give the answer as Vm


and option (c) will give the answer as V1. Both of
these cannot be the answers as Vm and V1 are the
amount of volume filled.
Let m = 2. So, option (b) will give the answer as 2
(1 V2) and option (d) will give the answer as
2(1 V1). Now consider option (b).
Actual empty volume > 2(1 V2). Therefore, for this
situation m(1 V1) is the only possible answer.

1
Let m = 1 and n = 1. Option (a) gives the answer as
4
and option (d) gives the answer as greatest integer

14. b

CAT 2001
15. d

x > 5, y < 1
Use answer choices.
Take x = 6, y = 6. We see none of the statements
(1, 2 and 3) is true. Hence the correct option is (d).

16. d

Let y = n3 7n2 + 11n 5


At n = 1, y = 0
(n 1) (n2 6n + 5)
= (n 1)2 (n 5)
Now (n 1)2 is always positive.
Now for n < 5 the expression gives a negative quantity.
Therefore, the least value of n will be 6.
Hence m = 6.

17. c

Let x be the number of males in Mota Hazri.


Chota Hazri
Mota Hazri
Males
x 4522
x
Females
2(x 4522)
x + 4020
x + 4020 2(x 4522) = 2910 x = 10154
Number of males in Chota Hazri = 10154 4522
= 5632

18. b

Let the number of students in classes X, Y and Z be a,


b and c respectively. Then
Total of X = 83a
Total of Y = 76b
Total of Z = 85c

1
. So, both of these cannot be
2
the answer. Option (b) gives the answer as smallest

less than or equal to

1
and option (c)
2
gives the answer as 1. But the actual answer can be
greater than 1 as the volume of the vessel is 2 l.
Hence, (b) is the answer.
integer greater than or equal to

CAT 2000
9. b

10. d

The data is not linear. So check (b).


Let the equation be y = a + bx + cx2.
Putting the values of x and y, we get the following
result.
4 = a + b + c, 8 = a + 2b + 4c and 14 = a + 3b + 9c.
Solving these, we get a = 2, b = 1 and c = 1.
So the equation is y = 2 + x + x2.

And

83a + 76b
= 79 , i.e. 4a = 3b
a+b

a1 = 1, a2 = 7, a3 = 19, a4 = 43.
The difference between successive terms is in series
6, 12, 24, 48, ..., i.e. they are in GP. Hence,

Also

76b + 85c
= 81 , i.e. 4c = 5b
b+c

299 1
rn 1
a100 = a1 + a
= 1+ 6
= 6 299 5
r 1
2

1
(
)

Hence, b =

11. c

12. c

The vehicle travels 19.5 km/L at the rate of 50 km/hr.

19.5
km/L at the rate of 70 km/hr
1.3

= 15 km/L.
The distance covered at 70 km/hr with 10 L = 10 15

Page 2

Use choices. The answer is (b), because x < 2 and


2 < 2y x < 2y.

5
5 4
5
4
a , c = b= a = a
4
4
3
3
3

Average of X, Y and Z =

1
1
1
1
+
+
+ ... +
1.3 3.5 5.7
19.21
1
1 1 1 1 1 1 1
1 1
1

= 1 + + + ...
2
3 23 5 25 7
2 19 21
1 1 (21 1) 20 10
=
=
=
=
2 42
42
42 21

So it should travel

13. b

Use the choices. If b = 1, then the factors are (x a)


(x2 + 1). This cannot yield 3 real roots.

83a + 76b + 85c


a+b+c

4
5
a + 85 a
978
3
3
= 81.5
=
4
5
12
a + a + a
3
3

83a + 76

19. a

Let the cost of 1 burger, 1 shake and 1 fries be x, y


and z.
Then
3x + 7y + z = 120
... (i)
4x + 10y + z = 164.5 ... (ii)
x + 3y = 44.5
... (iii) (ii i)
Multiplying (iii) by 4 and subtracting (ii) from it, we find
2y z = 13.5
...(iv)
Subtracting (iv) from (iii), we get x + y + z = 31.

Algebra

20. c

21. c

Let the 6th and the 7th terms be x and y.


Then 8th term = x + y
Also y2 x2 = 517
(y + x)(y x) = 517 = 47 11
So y + x = 47
y x = 11
Taking y = 29 and x = 18, we have 8th term = 47,
9th term = 47 + 29 = 76 and 10th term = 76 + 47 = 123.
x + y = 1 and x > 0 y > 0
Taking x = y =

BA =

MBA 2 =

24. a

1
, value of
2

c
= 12 c = 12 .
a
... (i)
Equation formed x2 7b + 12 = 0
Another boy gets the wrong roots (2, 3).

Sum of the roots =

For questions 22 and 23:


r1 + r2
r +r
, MBA 2 = 1 2 and
n1
n1 + n2

MBA1 =

b
= 5 b = 5 .
a

c
=6c =6.
a
Equation formed x2 5b + 6 = 0 ... (ii)
x2 + bx + c1 = 0

Product of the roots =

r
r1 n2
r
max 0, 2 1
+
n1 n1
n2 n1

b = 2 + 3
c = 6
Hence, x2 7x + 6 = 0
x2 6x x + 6 = 0
x(x 6) 1(x 6) = 0
(x 6)(x 1) = 0
x = 6, 1
Hence, the actual roots = (6, 1).

From BA and MBA 2, we get BA MBA 2 because


n1 + n2 n1 .

From BA and MBA 1, we get BA MBA1 because


r
r1 r2
r
r
n
r
+
1 + 2 2 max 0, 2 1 .
n1 n1 n1 n1 r2
n2 n1

Now from MBA1 and MBA2, we get

Alternate method:
Since constant = 6[3 2] and coefficient of
x = [4x 3x] = 7
Since quadratic equation is
x2 (Sum of roots)x + Product of roots = 0 or
x2 7x + 6 = 0
Solving the equation (x 6)(x 1) = 0 or x = (6, 1).

r
r1 r2 n2
r
r1
r2
max 0, 2 1
+

+
n1 n1 r2
n
n
n1 + n2 .
2
1 n1 + n2

22. d

b
= 7 b = 7 .
a

Product of the roots =

25 25 25
+
=
4
4
2

It can be easily verified as it is the least value among


options.

BA =

Equation of quadratic equation is


ax2 + bx + c = 0
x2 + bx + c = 0
First roots = (4, 3)
Sum of the roots =

2
2

1
1

1
1

x + x + y + y = 2 + + 2 +

2
2

r1 + r2
50n1 + 45
5
=
= 50
n1 + n2
n1 + 1
n +1

Hence, BA will increase, MBA2 will decrease.

r1 + r2 50n1 + 45
45
=
= 50 +
> 50
n1
n1
n1

From the above information, BA MBA1 MBA 2


None of these is the right answer.

CAT 2002
23. b

BA = 50 where there is no incomplete innings means r2


= n2 = 0

MBA1 =

r1
= 50
n1

r
r1 n2
r
max 0, 2 1
+
n1 n1
n
n
2

= 50 +

45
1

50
max 0,
n1
1

25. 2

1+ y
1+ x
f(x) + f(y) = log 1 x + log 1 y

(1 + x) (1 + y)
= log

(1 x)(1 y)
1 + x + y + xy
= log

1 + xy x y

= 50 + 0 = 50

Algebra

Page 3

1 + xy + x + y
= log

1 + xy (x + y)

S1(1 x) = 2 +

x
1 x

x + y
1+

1 + xy
= log

x + y
1 1 + xy

S(1 x 2 ) = 2 +

x S= 2x
(1 x)3
1 x

29. 3

(x 2 + 4y 2 4yx) + z2 + y 2 2yz = 0

x+y
= f

1 + xy
26. 4

27. 3

(x 2y)2 + (z y)2 = 0
It can be true only if x = 2y and z = y

x0 = x
x1 = x
x2 = x
x3 = x
x4 = x
x5 = x
x6 = x
..
Choices (1), (2), (3) are incorrect.

30. 2

31. 4

2x + 2y 2 R(x,y) 2x + 2y a 2 R a

xy + (y + x)(5 x y) = 3
x 2 + y 2 + xy 5x 5y + 3 = 0
y + (x 5) y + x 5x + 3 = 0
As it is given that y is a real number, the discriminant
for above equation must be greater than or equal to
zero.
2

Hence, (x 5)2 4(x 2 5x + 3) 0

32. 4

Let the largest piece = 3x


Middle = x
Shortest = 3x 23
or 3x + x + (3x 23) = 40
or x = 9
or the shortest piece = 3(9) 23 = 4
Check choices:
The shortest piece has to be < 20 cm.
27 is wrong choice.
The largest piece is a multiple of 3.
Or (23 + Shortest) should be a multiple of 3.
Answer = 4 cm (Among other choices)

34. 3

If p = q = r = 1, then expression = 1
Check the choice only, one choice gives the value of
expression = 1.

3x 10x 13 0
3x2 13x + 3x 13 0
13
x 1,
3

Coefficient of xn =

13
.
3

1
(n + 1)(n + 4)
2

S = 2 + 5x + 9x + 14x + ....
2

A2
B2
+
= 1 A 2 (x 1) + B2 x = x 2 x
x
x 1
This is a quadratic equation.
Hence, number of roots = 2 or 1 (1 in the case when
both roots are equal).

33. 3

28. 1

x 1 [x] x

Therefore, L R
Note: Choice (2) is wrong, otherwise choice (1) and
choice (3) are also not correct. Choose the numbers
to check.

xy + (y + x)z = 3

Largest value that x can have is

Arithmetic mean is more by 1.8 means sum is more by


18. So ba ab = 18
b > a because sum has gone up, e.g. 31 13 = 18
Hence, b a = 2

2x + 2y 3 L(x,y) 2x + 2y a 3 L a

xy + yz + zx = 3

x 2 + 5y 2 + z2 = 4yx + 2yz

xS = 2x + 5x 2 + .....
S(1 x) = 2 + 3x + 4x 2 + 5x 3 + ....

Let S1 = S(1 x) S1 = 2 + 3x + 4x 2 + ...


xS1 = 2x + 3x 2 + ...
S1(1 x) = 2 + x + x 2 + ....

Page 4

CAT 2003 (Leak)


35. 3

2x x 1 = 0
2x 1 = x
If we put x = 0, then this is satisfied and if we put
x = 1, then also this is satisfied.
Now we put x = 2, then this is not valid.

Algebra

36. 2

For the curves to intersect, log10 x = x1

Hence the minimum value of y is attained at x = 2.5

1
or x x = 10
x
This is possible for only one value of x (2 < x < 3).
Thus, log10 x =

37. 1

38. 1

39. 3

40. 4

41. 2

It is given that p + q + r 0 , if we consider the first


option, and multiply the first equation by 5, second by
2 and third by 1, we see that the coefficients of x,
y and z all add up-to zero.
Thus, 5p 2q r = 0
No other option satisfies this.
Let 'x' be the number of standard bags and 'y' be the
number of deluxe bags.
Thus, 4x + 5y 700 and 6x + 10y 1250
Among the choices, (3) and (4) do not satisfy the
second equation.
Choice (2) is eliminated as, in order to maximize profits
the number of deluxe bags should be higher than the
number of standard bags because the profit margin is
higher in a deluxe bag.

Alternate method:
At x = 2,
f(x) = 2.1
At x = 2.5, f(x) = 1.6
At x = 3.6, f(x) = 2.7
Hence, at x = 2.5, f(x) will be minimum.
42. 2

Solution can be found using Statement A as we know

1
1
and ).
2
2

both the roots for the equation (viz.


Also statement B is sufficient.
Since ratio of c and b = 1, c = b.

Thus the equation = 4x2 + bx + b = 0. Since x =

one of the roots, substituting we get 1

1
is
2

b
+ b = 0 or
2

b = 2. Thus c = 2.
43. 1

Let the 1st term be a and common difference be d


then we have 3rd term = a + 2d
15th term = a + 14d
6th term = a + 5d
11th term = a + 10d
13th term = a + 12d
Since sum of 3rd and 15th term = sum of 6th, 11th and
13th term, therefore we have
2a + 16d = 3a + 27d
a + 11d = 0
Which is the 12th term.

Both the series are infinitely diminishing series.


For the first series: First term =

For the second series: First term =

and r =

a2

a2

1
1
and r =
a
a2

The sum of the first series =

We can see that x + 2 is an increasing function and


5 x is a decreasing function. This system of equation
will have smallest value at the point of intersection of
the two. i.e. 5 x = x + 2 or x = 1.5.
Thus smallest value of g(x) = 3.5

a2 = 1
1
a2 1
1
2
a

The sum of the second series =

1
a
1

1
a2

a
2

a 1

Now, from the first statement, the relation can be


anything (depending on whether a is positive or
negative).
But the second statement tells us, 4a2 4a + 1 = 0 or

Case 1: If x < 2, then y = 2 x + 2.5 x + 3.6 x


= 8.1 3x.
This will be least if x is highest i.e. just less than 2.
In this case y will be just more than 2.1

1
. For this value of a, the sum of second series
2
will always be greater than that of the first.

a=
Case 2: If 2 x < 2.5 , then y = x 2 + 2.5 x +
3.6 x = 4.1 x
Again, this will be least if x is the highest i.e. just less
than 2.5. In this case y will be just more than 1.6.
Case 3: If 2.5 x < 3.6 , then y = x 2 + x 2.5 + 3.6
x = x 0.9
This will be least if x is least i.e. x = 2.5.
Case 4: If x 3.6 , then
y = x 2 + x 2.5 + x 3.6 = 3x 8.1
The minimum value of this will be at x = 3.6 and y = 2.7

Algebra

44. 4

The number of terms of the series forms the sum of


first n natural numbers i.e.

n(n + 1) .
2
Thus the first 23 letters will account for the first
23 24
= 276 terms of the series.
2
The 288th term will be the 24th letter which is x.

Page 5

45. 4

46. 2

47. 3

p + q = 2 and pq = 1
(p + q)2 = p2 + q2 + 2pq,
Thus ( 2)2 = p2 + q2 + 2( 1)
p2 + q2 = 2 4 + 4 + 2 + 2
p2 + q2 = 2 2 + 6
p2 + q2 = 2 2 + 1 + 5
p2 + q2 = ( 1)2 + 5
Thus, minimum value of p2 + q2 is 5.
(a + b + c + d)2 = (4m + 1)2
Thus, a2 + b2 + c2 + d2 + 2(ab + ac + ad + bc + bd + cd)
= 16m2 + 8m + 1
a2 + b2 + c2 + d2 will have the minimum value if (ab + ac
+ ad + bc + bd + cd) is the maximum.
This is possible if a = b = c = d = (m + 0.25) since a
+ b + c + d = 4m + 1
In that case 2((ab + ac + ad + bc + bd + cd)
= 12(m + 0.25)2 = 12m2 + 6m + 0.75
Thus, the minimum value of a2 + b2 + c2 + d2
= (16m2 + 8m + 1) 2(ab + ac + ad + bc + bd + cd)
= (16m2 + 8m + 1) (12m2 + 6m + 0.75)
= 4m2 + 2m + 0.25
Since it is an integer, the actual minimum value
= 4m2 + 2m + 1

negative value. In other words, vz has to be 2 and u


has to be 0.5.

vz
2
=
= 4.
0.5
u
For us to get the maximum value, vz has to be the
smallest negative value and u has to be the highest
negative value. Thus, vz has to be 2 and u has to be
0.5.

Hence the minimum value of

Hence the maximum value of


50. 2

GRRRRR, RGRRRR, RRGRRR, RRRGRR, RRRRGR,


RRRRRG
GGRRRR, RGGRRR, RRGGRR, RRRGGR, RRRRGG
GGGRRR, RGGGRR, RRGGGR, RRRGGG
GGGGRR, RGGGGR, RRGGGG
GGGGGR, RGGGGG
GGGGGG
Hence 21 ways.

51. 4

When we substitute two values of x in the above


curves, at x = 2 we get
y = 8 + 4 + 5 = 1
y= 42+5=7
Hence at x = 2 the curves do not intersect.
At x = 2, y1 = 17 and y2 = 11
At x = 1, y1 = 5 and 2 and y2 = 5
When x = 0, y1 = 5 and y2 = 5
And at x = 1, y1 = 7 and y2 = 7
Therefore, the two curves meet thrice when x = 1, 0
and 1.

52. 1

Let us say there are only 3 questions. Thus there are


231 = 4 students who have done 1 or more questions
wrongly, 232 = 2 students who have done 2 or more
questions wrongly and 233 = 1 student who must
have done all 3 wrongly. Thus total number of wrong
answers = 4 + 2 + 1 = 7 = 23 1 = 2n 1.
In our question, the total number of wrong answers
= 4095 = 212 1. Thus n = 12.

53. 3

Here x, y, z are distinct positive real number

Assume the number of horizontal layers in the pile be


n.
So

n(n + 1)
= 8436
2

1
[ n2 + n] = 8436
2

n(n + 1) (2n + 1) n(n + 1)


+
= 8436
12
4

2n + 4
n(n + 1)
= 8436
12

n(n + 1)(n + 2)
= 8436
6

n(n + 1) (n + 2) = 36 37 38
So n = 36

48. 3

2
y5
a
=
Using log a log b = log ,
, where
b y 5 y 3.5

y = 2x
Solving we get y = 4 or 8 i.e. x = 2 or 3. It cannot be 2
as log of negative number is not defined (see the
second expression).
49. 2

u is always negative. Hence, for us to have a minimum

vz
, vz should be positive. Also for the least
value of
u
value, the numerator has to be the maximum positive
value and the denominator has to be the smallest

Page 6

vz
2
=
= 4.
u
0.5

So
=

x 2 (y + z) + y 2 (x + 2) + z2 (x + y)
xyz

x x y y z z
+ + + + +
y z x z x y

x y y z z x
= + + + + + [We know that
y x z y x z

a b
+ > 2 if a and b are distinct numbers
b a
>2+2+2
>6

Algebra

54. 1

The least number of edges will be when one point is


connected to each of the other 11 points, giving a total
of 11 lines. One can move from any point to any other
point via the common point. The maximum edges will
be when a line exists between any two points. Two
points can be selected from 12 points in 12C2 i.e. 66
lines.

log10 x =

log10 x

1
2
log10 x =
2
log10 x

(log10 x ) = 4
2

CAT 2003 (Re-TEST)


55. 2

log10 100

log10 x = 2

Consider first zone. The number of telephone lines


can be shown a follows.

log10 x = 2 or log10 x = 2
102 = x or 10 2 = x

x = 100 or x =
= 9 lin es

58. 2
Therefore, total number of lines required for internal
connections in each zone = 9 4 = 36 lines.
Now consider the connection between any two
zones.

1
log3 M + 3log3 N = 1 + log0.008 5
3
log3 (M1/ 3N3 ) = 1 +

log3 (M1/ 3N3 ) = 1

log3 (M1/ 3N3 ) = 1

Each town in first zone can be connected to three


towns in the second zone.
Therefore, the lines required = 3 3 = 9
Therefore, total number of lines required for connecting
towns of different zones = 4C2 9 = 6 9 = 54
Therefore, total number of lines in all = 54 + 36 = 90
56. 2

59. 3

b2 4ac 0

b2 4a

For a = 1, 4a = 4, b = 2, 3, 4
a = 2, 4a = 8, b = 3, 4
a = 3, 4a = 12, b = 4
a = 4, 4a = 16, b = 4

Number of equations possible = 7.


57. 2

log10 x log10 x = 2logx 10


x
log10
= logx 100
x

Algebra

(log10 log2 )
log8 log1000

(1 log2 )

3 (1 log2 )

1 2
=
3 3

M1/3N3 = 32/3
MN9 = 32 N9 = 9/M.

ax2 + bx + 1 = 0
For real roots

b2 4a(1) 0

1
100

60. 4

5x + 19y = 64
We see that if y =1, we get an integer solution for
x = 9, now if y changes (increases or decreases) by
5x will change (decrease or increase) by 19.
Looking at the options, if x = 256, we get y = 64.
Using these values we see options (1), (2) and (4)
are eliminated and also that these exists a solution for
250 < x 300.

m2
m3
+ log
Sum of log m + log
n
n2

+ L n terms such

problem must be solved by taking the value of number


of terms. Lets say 2 and check the given option. If we
look at the sum of 2 terms of the given series it comes

m3

= log
n
n
n

Now look at the option and put number of terms as 2,


only option (4) validates the above mentioned answer.
out to be log m + log

m2

log

m m2

m(n +1) 2
m3
m3
log
As log (n 1) log
n
n

Page 7

61. 2

xyz = 4
yx=zy
2y = x + z
y is the AM of x, y, z.
2

= 2, = 1
But the above values of do not satisfy any of the
equations. Thus, no root is common.
1

Also 3 xyz = 4 3 3 xyz = 2 3

64. 3

AM GM

1
1
< x3+
n
n
Put n = 1
1

0<x4

2
y 23
2

65. 4

Therefore, the minimum value of y is 2 3 .

36 n 72
x=

62. 3

Let S = 1 +

4 9 16 25
+
+
+
7 7 2 73 7 4

1
1 4
9 16
S= +
+
+
7
7 7 2 73 7 4
(i) (ii) gives,

1
3 5
7
9

S 1 = 1 + +
+
+
7
7 7 2 73 7 4

1
1 1 3
5
7

S 1 = + 2 + 3 + 4
7
7
7
7
7
7

n2 + 2 n(n + 4) + 16
n+4 n +4

(i)

Put x = 36.

(ii)

(36)2 + 2 6 40 + 16
36 + 24 + 4
Which is least value of n = 28

(iii)

x=

66. 4

13x + 1 < 2z and z + 3 = 5y 2


13x + 1 < 2 (5y 2 3)

(iv)

13x + 1 < 10y 2 6

(iii) (iv) gives,

13x + 7 < 10y 2 put x = 1

1 1
1
2 2
2
2

S 1 S 1 = 1 + +
L
+
+
7 7
7
7 72 73 74

1
1
2
1 1

S 1 1 = 1 + 1 + +
+ L
7
7
7 7 72

20 < 10y 2

y2 > 2

y2 > 2

(y 2 2) > 0

X=1

1
2
1

S 1 = 1 +
1
7
7

1
7
2

2 7
6
S = 1+
7 6
7

36
1
S
= 1+
49
3

S =
S=
63. 1

49 4

36 3

49
27

Let is the common root.


3 + 3 2 + 4 + 5 = 0

3 + 23 + 7 + 3 = 0

67. 2

y
2

x = |a| b
Now a xb = a ( |a| b) b
= a + |a| b2
a xb = a + ab2 a 0 OR a xb
= a ab2 a < 0
= a(1 + b2) = a(1 b2)
Consider first case:
As a 0 and |b| 1, therefore (1 + b2) is positive.
a (1 + b2) 0
a xb 0
Consider second case.
As a < 0 and |b| 1, therefore (1 b2) 0
a (1 b2) 0 (Since ve -ve = +ve and 1 b2 can
be zero also), i.e. a xb 0
Therefore, in both cases a xb 0.

2 3 + 2 = 0

Page 8

Algebra

68. 1

74. 1

g2 = g g = h
g3 = g2 g = h g = f
g4 = g3 g = f g = e

n=4
69. 4

f [f {f (f f )}]

= f [f {f h}]]
= f [f e}]
= f [f ]]
=h
70. 1

75. 3

e8 = e 2 e 2 e 2

71. 4

a
b
c
=
=
=r
b+c c +a a+b
then there are only two possibilities.
(i)
If

If a + b + c 0, then

= eee
=e
If we observe a
a10 = a

We have
(1) 1010 < n < 1011
(2) Sum of the digits for 'n' = 2
Clearly(n)min = 10000000001 (1 followed by 9 zeros and
finally 1)
Obviously, we can form 10 such numbers by shifting
'1' by one place from right to left again and again.
Again, there is another possibility for 'n'
n = 20000000000
So finally : No. of different values for n = 10 + 1 = 11
ans.

a
b
c
a+b+c
=
=
=
b + c c + a a + b (b + c) + (c + a) + (a + b)

anything = a

a+b+c
1
=
2(a + b + c) 2

{a10 (f 10 g9 )} e8

= ae
=e

(ii)
If a+ b + c = 0, then
b+c=a
c+a=b
a+b=c

It will go by elimination.
9 7 = 2 is even, therefore option (1) not possible.
2 9 = 18 is even, therefore option (2) not possible.

a
a
=
= 1
b + c (a)

Hence

3 + 9 12
=
= 4 is even, therefore option (3) is not
3
3
possible.
The correct option is (4).

b
c
=
= 1
c +a a+b
Therefore option (3) is the correct one 1/2 or 1
Similarly,

CAT 2004
72. 1

Given

76. 4

y=

t1 + t 2 + ... + t11 = t1 + t 2 + .... + t19 ( for an A.P.)


11
19
[2a + (11 1)d] = 2 [2a + (19 1)d]
2
22 a + 110d = 28 a + 342 d
16 a + 232 d = 0
2a + 29 d = 0

30
[2a + (30 1)d] = 0
2

S30terms = 0
73. 2

1
2+

y=

1
3+y

3+y
7 + 2y

2y 2 + 6y 3 = 0

y=
=

6 36 + 24
4

6 60 3 15
=
4
2

We have
f(0) = 03 4(0) + p = p

Since 'y' is a +ve number, therefore:

f(1) = 13 4(1) + p = p 3

y=

15 3
ans.
2

If P and P 3 are of opp. signs then p(p 3) < 0


Hence 0 < p < 3.

Algebra

Page 9

77. 4

When a > 0, b < 0,


ax2 and b |x| are non negative for all x,
i.e. ax2 b|x| 0
ax2 b |x| is minimum at x = 0 when a > 0, b < 0.

82. 1

Since Group (B) contains 23 questions, the marks


associated with this group are 46.
Now check for option (1). If Group (C ) has one
question, then marks associated with this group will
be 3. This means that the cumulative marks for these
two groups taken together will be 49. Since total
number of questions are 100, Group (A) will have 76
questions, the corresponding weightage being 76
marks. This satisfies all conditions and hence is the
correct option. It can be easily observed that no other
option will fit the bill.

83. 3

Since Group (C) contains 8 questions, the


corresponding weightage will be 24 marks. This figure
should be less than or equal to 20% of the total marks.
Check from the options . Option (3) provides 13 or 14
questions in Group (B), with a corresponding
weightage of 26 or 28 marks. This means that number
of questions in Group (A) will either be 79 or 78 and
will satisfy the desired requirement.

78. 4
Family
1
II
III

Adults
0, 1, 2
0, 1, 2
0, 1, 2

Children
3, 4, 5, .
3, 4, 5, .
3, 4, 5, .

As per the question, we need to satisfy three


conditions namely:
1. Adults (A) > Boys (B)
2. Boys (B) > Girls (G)
3. Girls (G) > Families (F)
Clearly, if the number of families is 2, maximum number
of adults can only be 4. Now, for the second condition
to be satisfied, every family should have atleast two
boys and one girl each. This will result in noncompliance with the first condition because adults will
be equal to boys. If we consider the same conditions
for 3 families, then all three conditions will be satisfied.

CAT 2005
65

79. 3

Given equation is x + y = xy
xy x y + 1 = 1
(x 1)(y 1) = 1

84. 4

u = (log2x)2 6log2x + 12
xu = 256
Let log2x = y x = 2y
xu = 28 uy = 8 u =

8
y

8
= y 2 6y + 12 y3 6y 2 + 12y 8 = 0
y
(y 2)3 = 0 y = 2

x = 4, u = 4

Page 10

+ (30 1)

65
64

3065 3065 1
30

64

3064 + 3064 1
30

65

1
1 1

3665
30
or R =

64
3064
1
1
1
+


30

Given a1 = 81.33; a2 = 19
Also:
aj = aj1 aj2, for j 3
a3 = a2 a1 = 100.33
a4 = a3 a2 = 81.33
a5 = a4 a3 = 19
a6 = a5 a4 = +100.33
a7 = a6 a5 = +81.33
a8 = a7 a6 = 19

1 (0.96 )65

R = 30
64
1 + (0.96 )

1 (0.96 )

65

Clearly onwards there is a cycle of 6 and the sum of


terms in every such cycle = 0. Therefore, when we
add a1, a2, a3... upto a6002, we will eventually be left with
a1 + a2 only i.e. 81.33 19 = 62. 33.
81. 2

(30 1)

64

30
30

x 1 = 1& y 1 = 1or x 1 = 1& y 1 = 1


Clearly (0, 0) and (2, 2) are the only pairs that will
satisfy the equation.

80. 3

R=

65

in

1 + (0.96 )

64

Nr is only slightly less then 1.


& Dr is only slightly more than 1. R is slightly less than
36 but certainly greater than 1.
85. 4

If p = 1! = 1
Then p + 2 = 3 when divided by 2! remainder will be 1.
If p = 1! + 2 2! = 5
Then p + 2 = 7 when divided by 3! remainder is still 1.
Hence p = 1! + (2 2!) + (3 3!) + + (10 10!) when
divided by 11! leaves remainder 1
Alternative method:
P = 1 + 2.2! + 3.3!+ .10.10!
= (2 1)1! + ( 3 1)2! + (4 1)3! + .(11 1)10!
=2! 1! + 3! 2! + .. 11! 10! = 1 + 11!
Hence the remainder is 1.

Algebra

86. 3

87. 4

a1 = 1,
an+1 3an + 2 = 4n
an+1 = 3an + 4n 2
when n = 2 then a2 = 3 + 4 2 = 5
when n = 3 then a3 = 3 5 + 4 2 2 = 21
from the options, we get an idea that a n can be
expressed in a combination of some power of
3 & some multiple of 100.
(1) 399 200; tells us that an could be: 3n1 2 n;
but it does not fit a1 or a2 or a3
(2) 399 + 200; tells us that an could be: 3n1 + 2 n;
again, not valid for a1, a2 etc.
(3) 3100 200; tells 3n 2n: valid for all a1, a2, a3.
(4) 3100 + 200; tells 3n + 2n: again not valid.
so, (3) is the correct answer.

= logx x logx y + logy y logy x


= 2 logx y logy x

)(

2 x1/ 3 1 8 x 1
92. 4

Let number of elements in progression be n, then

1000 = 1 + (n 1) d (n 1) d = 999 = 33 37
Possible values of d = 3, 37, 9, 111, 27, 333, 999
Hence 7 progressions.
From the graph of (y - x) Vs. (y + x), it is obvious that
inclination is more than 45.
Slope of line =

yx
= tan(45 + );
y+x

y x 1 + tan
=
y + x 1 tan

By componendo-dividendo,

y x 1 + tan
=
which
y + x 1 tan

Which can never be positive, out of given option it


cant assume a value of +1. So (4) is ans.

is nothing but the slope of the line that shows the


graph of y Vs. x.
And as 0 < < 45, absolute value of tan is less
than 1.

x = 4 + 4 x x2 = 4 + 4 x

1
is negative and also, greater than 1.
tan
The slope of the graph y Vs. x must be negative
and greater than 1. Accordingly, only option (4)
satisfies.
You can also try by putting the values of (y + x) =
2(say) and (y - x) = 4(anything more than 2 for that
matter). You can solve for values of y and x and
cross check with the given options.
Alternate method:
In the normal X-Y coordinate plane the X-axis
corresponds to y = 0
And Y-axis corresponds to x = 0
y + x = 0 and y - x = 0 are perpendicular lines on this
plane.

( x2 4 ) =

4x

Now put the values from options.


Only 3rd option satisfies the condition.
89. 4

1/ 3
1 x1/ 3 + 2 0
x

Let, t = logx y

88. 3

x 2 / 3 + x1/ 3 2 0
x 2 / 3 + 2x1/ 3 x1/ 3 2 0

93. 4

x
y
P = logx + logy
x
y

1
1

p = 2 t = t
t
t

91. 1

There are two equations to be formed 40 m + 50 f


= 1000
250 m + 300 f + 40 15 m + 50 10 f = A
850 m + 8000 f = A
m and f are the number of males and females A is
amount paid by the employer.
Then the possible values of f = 8, 9, 10, 11, 12
If f = 8, M = 15
If f = 9, 10, 11 then m will not be an integer while f = 12
then m will be 10.
By putting f = 8 and m = 15, A = 18800. When f = 12 and
m = 10 then A = 18100
Therefore the number of males will be 10.

y
y+x=0
(Y X ) a xis
(yx)=0

CAT 2006
(Y + X ) a xis

90. 1

t3t4t5....t53

3 4 5
51 52 53
34
2
.......

=
=
5 6 7
53 54 55 54 55 495
Hence option (1).

Algebra

Page 11

And y-x = 0 is the axis Y+X and y+x = 0 is the axis


Y-X
So, the dotted line is the graph drawn in the question.
When you observe w.r.t to X-axis it looks like

So, ab a3b3 = 0

2 2
Or, a b 1 a b = 0

ab = 1
Only option (5) does not satisfy.
Hence (5).

96. 5

Equation (ii) can be written as

40.3x 90.2y = 8 (81)

1/ 5

(22 )0.3x (32 )0.2y = 8 (81)1/ 5

94. 2

20.6x 30.4y = 23 (3 4 )1/ 5 = 23 3 4 / 5

2x + y = 40
xy
y = 40 2x
Values of x and y that satisfy the equation

0.6x = 3 x = 5
and 0.4y =

4
5

y =2
x

38

36

34

13

14

If we put the values of x and y in first equation these


values satisfy the first equation also.
So the answer is x = 5, y = 2
Hence, option (5)
97. 5

y
x
+
=1
1 1/ 2
Similarly y + 4x = 3

13 values of (x, y) satisfy the equation such that x


y
95. 5

logxy = a.logzy = b.logzx = a b


a=

logxy
logzy

and b =

ab =

f(x) = max (2x + 1, 3 - 4x)


So, the two equations are y = 2x + 1 and y = 3 - 4x
y 2x = 1

logxy

x=

logzy

logx
y

logzy logzy
logxy

y
x
+
=1
3 3/4
Their point of intersection would be
2x + 1 = 3 - 4x
6x = 2
1
3

y
y=2 x+ 1
(0 ,3)

logx
k
logy
k
logy
k
logz
k

logx
= k
logy
k

Page 12

logx
k
logy
k
logy
k
logz
k
3

(1 /3,5/3 )

{For some base

y=3 4 x
x
(-1 /2,0)

(0 ,3/4)

3
= logx = (ab )3
y

( )

Algebra

So, when x

1
then f(x)max = 3 4x
3

And when x

1
then f(x)max = 2x + 1
3

Hence the min. of this would be at x =

values of x and y in that order are (7, 5); (17, 4); (27,
3);
(37, 2); (47, 1) and (57, 0).
For z = 2: x + 10y = 7
There is only one integer value of x and y that
satisfies the equation x + 10y = 7 in that order is
(7, 0).
Therefore total number of ways in which you can
pay a bill of 107 Misos = 11 + 6 + 1 = 18

1
3

5
3

i.e. y =

100. 5

Alternative method:
As f(x) = max (2x + 1, 3 - 4x)
We know that f(x) would be min at the point of
intersection of these curves
i.e. 2x + 1 = 3 - 4x
6x = 2
i.e. x =

1
3

Hence min f(x) is

5
3

1 4 1
+ = , n < 60
m n 12

1
1 4 n 48
=
=
m 12 n
12n

12n
n 48
Positive integral values of m for odd integral values of
n are for n = 49, 51 and 57.
Therefore, there are 3 integral pairs of values of m
and n that satisfy the given equation.
m=

CAT 2007
98. 1

f(1) + f(2) + f(3) + . + f(n) = n2 f(n) , f(1) = 3600.


For n = 2:
f(1)

f(1)
+ f(2) = 22 f(2) f(2) =
(22 1)

For n = 3:

3600 1 +
2 1
2

240 + 20b + 202 c

(240 + 60b + 602 c ) (240 + 40b + 402 c ) = 1


240 + 40b + 402 c

Similarly
f(9) = 3600

2 3 4 ... 8

)(

)(

Solving the above equations c =

) (

22 1 32 1 42 1 ... 92 1

Therefore, f(9) = 80
99. 3

Let the number of currency 1 Miso, 10 Misos and 50


Misos be x, y and z respectively.
x + 10y + 50z = 107
Now the possible values of z could be 0, 1 and 2.
For z = 0: x + 10y = 107
Number of integral pairs of values of x and y that
satisfy the equation x + 10y = 107 will be 11. These
values of x and y in that order are
(7, 10); (17, 9); (27, 8)
(107, 0).
For z = 1: x + 10y = 57
Number of integral pairs of values of x and y that
satisfy the equation x + 10y = 57 will be 6. These

Algebra

and

22 1

f(3) = 3600
22 1 32 1

102. 2 Using the given data

(240 + 40b + 402 c ) (240 + 20b + 202 c ) = 2

2
+ f(3) = 3 f(3)

101. 1 Using A: x = 30, y = 30 and z = 29 will give the


minimum value.
Using B: Nothing specific can be said about the relation
between x, y and z.
Hence option (1) is correct choice.

1
and b = 10
10

So, cost for producing x units = 240 + 10x +

x2
10

Profit earned from x units

x2
x2
= 20x
240
= 30x 240 + 10x +

10
10

For maximum profit 20 =

2x
10

So, x = 100.

Page 13

CAT 2008

x2
240
103. 4 Profit, in rupees = 20x
10
For x = 100
Profit, in rupees = 760

107. 2 x3 ax2 + bx c = 0
Let the roots of the above cubic equation be
( 1), , ( + 1)
( 1) + ( + 1) + ( + 1) ( 1) = b
2 + 2 + + 2 1 = b 32 1 = b

104. 2 Let f(x) = ax 2 + bx + c


At x = 1, f(1) = a + b + c = 3
At x = 0, f(0) = c = 1
The maximum of the function f(x) is attained at

b
a2
=1=
x=
2a
2a
a = 2 and b = 4

Therefore f(x) = 2x2 + 4x + 1


Therefore f(10) = 159
For questions 105 and 106:
Using the given expressions

Thus, the minimum possible value of b will be equal


to 1 and this value is attained at = 0.
108. 2 Given that f(x) = ax2 + bx + c
Also, f(5) = 3f(2) f(5) + 3f(2) = 0
(25a + 5b + c) + 3(4a + 2b + c) = 0
37a + 11b + 4c = 0
(i)
Also, as 3 is a root of f(x) = 0, therefore f(3) = 0.
Therefore, 9a + 3b + c = 0
(ii)
Using equation (i) and (ii), we get that a = b
Therefore, c = 12a
f(x) = a(x2 + x 12) = a(x + 4) (x 3)
Therefore, the other root of f(x) = 0 is 4.

a1 = p

b1 = q

a2 = pq

b2 = q2

109. 5 f(x) = a(x2 + x 12)


Therefore, the value of a + b + c cannot be uniquely
determined.

a3 = p 2 q

b3 = pq2

110. 3 Total number of terms in the sequence 17, 21, 25

2 2

a4 = p q

b4 = pq3

417 17
+ 1 = 101 .
4
Total number of terms in the sequence 16, 21, 26
466 16
+ 1 = 91 .
466 is equal to
5
nth term of the first sequence = 4n + 13.
mth term of the second sequence = 5m + 11.
As per the information given in the question 4n + 13
= 5m + 11
5m 4n = 2.
Possible integral values of n that satisfy 5m = 2 + 4n
are (2, 7, 12 97)
Therefore, the total number of terms common in both
the sequences is 20.
417 is equal to

a5 = p3q2 b5 = p2q3
a6 = p3q3 b6 = p2q4

and so on
n n

105. 1

an + bn (n is even ) = p 2 q 2 + p 2
n1

= q (pq ) 2

q2

+1

(p + q)
n +1 n 1

106. 4

n 1 n +1

an + bn (n is odd ) = p 2 q 2 + p 2 q 2
n1

= (p + q )(pq ) 2

Substituting p =

1
2
and q =
3
3

n 1
2 2

an + bn =
9

Substituting n = 7, an + bn > 0.01


Substituting n = 9, an + bn < 0.01
Hence smallest value of n is 9

Page 14

111. 2

f(x).f(y) = f(xy)
Given, f(2) = 4
We can also write;
f(2) = f(2 1) = f(2) f(1)
OR f(1) 4 = 4
f(1) = 1
Now we can also write,
1

1
f(1) = f 2 = f(2) f
2

2
1 f(1) 1
OR f =
=
2 f(2) 4
1
1
OR f =
2 4

Algebra

112. 5 seed(n) function will eventually give the digit-sum of


any given number, n.
All the numbers n for which seed(n) = 9 will give the
remainder 0 when divided by 9.
For all positive integers n, n < 500, there are 55 multiples
of 9.

113. 1

S = 1+

1
12

+ 1+

22

Tn = 1 +

1
32

+ ... +

1+

1
20072

1
20082

1
(n + 1)2

n2 (n + 1)2

n2 + n + 1
n2 + n

2007

n =1

Algebra

n4 + 2n3 + 3n2 + 2n + 1

S=

1
n2

1
22

= 1+

Tn = 2007 +

1
n2 + n
2007

1
1
1

= 2008 2008
n
n
1
+

n =1

Page 15

You might also like